IASbaba’s Daily CSAT Practice Test
ARCHIVES
Daily CSAT Practice Test
Everyday 5 Questions from Aptitude, Logical Reasoning, and Reading Comprehension will be covered from Monday to Saturday.
Make the best use of the initiative. All the best!
To Know More about Ace the Prelims (ATP) 2021 – CLICK HERE
Important Note:
- Don’t forget to post your marks in the comment section. Also, let us know if you enjoyed today’s test 🙂
- After completing the 5 questions, click on ‘View Questions’ to check your score, time taken and solutions.
Test-summary
0 of 5 questions completed
Questions:
- 1
- 2
- 3
- 4
- 5
Information
To view Solutions, follow these instructions:
- Click on – ‘Start Test’ button
- Solve Questions
- Click on ‘Test Summary’ button
- Click on ‘Finish Test’ button
- Now click on ‘View Questions’ button – here you will see solutions and links.
You have already completed the test before. Hence you can not start it again.
Test is loading...
You must sign in or sign up to start the test.
You have to finish following test, to start this test:
Results
0 of 5 questions answered correctly
Your time:
Time has elapsed
You have scored 0 points out of 0 points, (0)
Average score |
|
Your score |
|
Categories
- Not categorized 0%
Pos. | Name | Entered on | Points | Result |
---|---|---|---|---|
Table is loading | ||||
No data available | ||||
- 1
- 2
- 3
- 4
- 5
- Answered
- Review
-
Question 1 of 5
1. Question
By selling 33 m of carpet, a man loses an amount equal to the selling price of 3 m of carpet. Find his gain or loss per cent.
Correct
Solution (a)
Loss = (CP of 33 m) – (SP of 33 m)
(SP of 3 m) = (CP of 33 m) – (SP of 33 m)
(SP of 33 m) + (SP of 3 m) = (CP of 33 m)
(SP of 36 m) = (CP of 33 m).
Let the CP of 1 m be Rs x.
Then, CP of 36 m = Rs 36x
SP of 36m = (CP of 33m) = Rs 33x.
Thus, CP = Rs 36x and SP = Rs 33x.
Since (CP) > (SP), there is a loss.
Loss = Rs (36x – 33x) = Rs 3x.
Loss% = [(loss/CP) × 100] %
= [(3x/36x) × 100] % = 25/3% = 8 (1/3) %.
Incorrect
Solution (a)
Loss = (CP of 33 m) – (SP of 33 m)
(SP of 3 m) = (CP of 33 m) – (SP of 33 m)
(SP of 33 m) + (SP of 3 m) = (CP of 33 m)
(SP of 36 m) = (CP of 33 m).
Let the CP of 1 m be Rs x.
Then, CP of 36 m = Rs 36x
SP of 36m = (CP of 33m) = Rs 33x.
Thus, CP = Rs 36x and SP = Rs 33x.
Since (CP) > (SP), there is a loss.
Loss = Rs (36x – 33x) = Rs 3x.
Loss% = [(loss/CP) × 100] %
= [(3x/36x) × 100] % = 25/3% = 8 (1/3) %.
-
Question 2 of 5
2. Question
The Vice Chancellor of a University wants to select a team of five member organizing committee for the upcoming convocation of the University to be held in March 2020. The committee members are to be selected from five shortlisted professors (Prof A, B, C, D and E) and four short listed students (P, Q, R and S). Some conditions for selection of the committee members are given below:
Prof A and S have to be together
P cannot be put with R
Prof D and Q cannot be together
Prof C and Prof E have to be selected
R cannot be selected with Prof B
If two members of the committee are students and Prof D is one of the members of the committee, who are the other committee members?
Correct
Solution (d)
In the paragraph it is given that,
Prof C and Prof E have to be selected.
Also, the question says Prof D is selected to be the committee member.
2 of the members are students thus; Prof A and Prof B cannot be selected.
Prof A and S have to be together. Thus, S cannot be selected.
Prof D and Q cannot go together. Thus, Q cannot be selected.
Thus, P and R will have to be selected. But, P cannot be put with R.
Thus, no team is possible. Hence, option d is correct.
Incorrect
Solution (d)
In the paragraph it is given that,
Prof C and Prof E have to be selected.
Also, the question says Prof D is selected to be the committee member.
2 of the members are students thus; Prof A and Prof B cannot be selected.
Prof A and S have to be together. Thus, S cannot be selected.
Prof D and Q cannot go together. Thus, Q cannot be selected.
Thus, P and R will have to be selected. But, P cannot be put with R.
Thus, no team is possible. Hence, option d is correct.
-
Question 3 of 5
3. Question
Read the below paragraph with the given conditions and answer the questions that follow.
The Vice Chancellor of a University wants to select a team of five member organizing committee for the upcoming convocation of the University to be held in March 2020. The committee members are to be selected from five shortlisted professors (Prof A, B, C, D and E) and four short listed students (P, Q, R and S). Some conditions for selection of the committee members are given below:
Prof A and S have to be together
P cannot be put with R
Prof D and Q cannot be together
Prof C and Prof E have to be selected
R cannot be selected with Prof B
In case Prof A and Prof C are members, who are the other members who cannot be selected for the committee?
Correct
Solution (d)
We are given that,
Prof A and S have to be together and Prof C and Prof E have to be selected as well
Thus, 4 of the committee members are Prof A, S, Prof C and Prof E.
We see that Prof E has to be selected for the committee and is present in all the options a, b and c.
Hence, the correct answer is option d.
Incorrect
Solution (d)
We are given that,
Prof A and S have to be together and Prof C and Prof E have to be selected as well
Thus, 4 of the committee members are Prof A, S, Prof C and Prof E.
We see that Prof E has to be selected for the committee and is present in all the options a, b and c.
Hence, the correct answer is option d.
-
Question 4 of 5
4. Question
The price of commodity X increases by 40 paise every year, whereas the price of commodity Y increases by 15 paise every year. If in 2011, the price of commodity X was Rs 4.20 and that of Y was Rs 6.30, in which year commodity X will cost 40 paise more than the commodity Y?
Correct
Solution (b)
Suppose commodity X will cost 40 paise more than Y after z years.
Then, (4.20 + 0.40z) – (6.30 + 0.15z) = 0.40
=> 0.25z = 0.40 + 2.10
=> z =2.50/0.25= 250/25 = 10.
=> X will cost 40 paise more than Y 10 years after 2011 i.e., 2021.
Incorrect
Solution (b)
Suppose commodity X will cost 40 paise more than Y after z years.
Then, (4.20 + 0.40z) – (6.30 + 0.15z) = 0.40
=> 0.25z = 0.40 + 2.10
=> z =2.50/0.25= 250/25 = 10.
=> X will cost 40 paise more than Y 10 years after 2011 i.e., 2021.
-
Question 5 of 5
5. Question
A person starts from point P towards East and moves 12 m to point Q. Then, he moves to his right for 8 m to point R. Again he moves to his right for 6 m to point S. Then, he moves 6 m towards North to point T. Finally from there he goes to his left for 6 m to point U. Which of the following three point would form a triangle whose all the angles are less than 90°?
Correct
Solution (a)
Using the information given we can represent it as
Therefore, three points Q, T and R, would form a triangle whose all the angles are less than 90°.
Incorrect
Solution (a)
Using the information given we can represent it as
Therefore, three points Q, T and R, would form a triangle whose all the angles are less than 90°.